If there are green balls in boxes 5 and 6, then which one of the following could be true?

zia305 on July 29, 2020

Explanation

Need help with game setup and this answer?

Replies
Create a free account to read and take part in forum discussions.

Already have an account? log in

shunhe on July 30, 2020

Hi @zia305,

Thanks for the question! So as for the setup of this game: the basic structure you should be working with is going to be six vertical lines, like

__
__
__
__
__
__

since that’ll mimic the boxes stacked on each other from 1 to 6 (remember that 1 is going to be the lowest box). And of course, the balls are going to be R, G, or W. ??First rule: R > W

Second rule:
All Rs
G

as in, we know that somewhere, there’s a G under all the Rs. This also lets us know that box 1 can’t have a red ball, so we can mark that one the vertical lines.

Third rule:

G
W +

In other words, there’s a white ball below a green ball somewhere.

Now we have 3 possible ball colors, but 6 possible boxes to put them in. So this should clue us in that we should be thinking about the numerical distributions of the colors.

We know there’s at least one green ball and at least one white ball from the second and third rules. Which means that there have to be at least two red balls based on the first rule. So we have 2 more balls that are of an unknown color. Well, either both of them can be red, one can be red and one can be green, one can be red and one can be white, or both of them can be green (they can’t both be white, because R > W).

So we should keep those distributions in mind; there are four possible distributions of numbers to balls in this game.

Now as for this specific question: we’re told here that there are green balls in boxes 5 and 6. Well, that means that there are at least two green balls, which eliminates 2 of the numerical distributions. But also note that if there are green boxes in the top two boxes, there has to be another green ball, since there has to be a green ball underneath all the red balls. So there are three green balls. Now we know that this is the scenario in which there’s 3 green balls, 2 red balls, and 1 white ball.

So knowing this, there’s two possibilities, since G has to be under all the Rs, and W has to be under G. We could have R R in boxes 2 and 3, or 3 and 4. So the first is

G
G?W
R
R
G

and the second is

G
G
R
R
G
W

and those are the only two possibilities. Immediately, we can see that (C) happens in the second possibility, and so it’s the correct answer in this could be true question.

Hope this helps! Feel free to ask any other questions that you might have.

zia305 on July 30, 2020

@shune Thanks so much. Also if you could help me with September 2016 logical game 4. It's a lost boys game type. If you could provide game setup and explanation for all the questions. The video explanations are not working and I have reached out to support but the issue haven't been resolved. If you could provide help with that please!!!